For a Pendulum: Knowing Acceleration Find Maximum Angle

In summary, the pendulum in a clock goes up and down (oscillates) at the same speed, meaning that when the pendulum reaches its maximum height it changes direction instantly.
  • #1
babaliaris
116
15
PS: By the way today I had exams in Physics and this problem was the first one I had to solve :p (unlucky) The question was to find the maximum angle θ that the pendulum can reach if we know that the magnitude of the acceleration is the same when the mass is located in the highest and the lowest point. The lowest point is when the mass passes exactly from the vertical position and the highest is when the angle θ(t) becomes maximum (which means that |V|=0).

And the problem is, if V does not change in speed, how can it become zero? How can the pendulum oscillate without the speed ever becoming zero at the maximum angle?

I will attach my solution which is exactly what I wrote in the exams today.

The problem description goes as follows:
"An object of mass m hangs from the roof with a rope of length L. The maximum angle that is between the rope and the vertical line (the one that is perpendicular to the roof where the rope is attached) is Θ. The magnitude of the acceleration is the same at the highest and lowest point of the path. Find θ in radians."

My solution, in short, was the following (note that when I say Θ(t) I don't mean the maximum angle, I just mean the function over time of the angle. in other words Θ = max{Θ(t)}):

Using everything I learned from this post, I found that
##a_{T} = gsin(θ(t))##
Also, I know ##a_{N} = \frac{|V|^2}{L}##
and then I found out that ##|a| = \sqrt{a_{T}^{2} + a_{N}^{2}} = \sqrt{g^{2}sin^{2}(θ(t)) + \frac{|V|^4}{L^{2}}}##

and then I said that ##|a1|= \sqrt{g^{2}sin^{2}(θ(t)) + 0} = gsin(θ(t)) ## since at the highest point the speed is zero and ##|a2| = \sqrt{g^{2}sin^{2}(θ(t)) + \frac{ |V_{max}|^4 }{L^2} }##

and if you say ##|a1| = |a2| <=>gsin(θ(t)) =\sqrt{g^{2}sin^{2}(θ(t)) + \frac{ |V_{max}|^4 }{L^2} } <=> g^{2}sin^{2}(θ(t)) = g^{2}sin^{2}(θ(t)) + \frac{|V_{max}|^4}{L^2} => V_{max} = 0## which is not correct.
 

Attachments

  • Exams Problem Pendulum.pdf
    507.3 KB · Views: 80
Last edited:
Physics news on Phys.org
  • #2
PS: O MY GOD, I just figured something out... Well if you see the pendulum in a clock, you see it going up and down (oscillating) at the same speed! This means that when the pendulum reaches its maximum height it changes direction instantly! This is why the speed never changes but the direction of V changes!

This is the wrong assumption that I did and used V = 0 at the highest point which is not correct!

I did what my tutor always told me not to do "Do not assume things based on your intuition. Only rely on the math and the data that was given".
 
  • #3
But now, how can I use the equation ##|a| = \sqrt{a_{T}^{2} + a_{N}^{2}} = \sqrt{g^{2}sin^{2}(θ(t)) + \frac{|V|^4}{L^{2}}}## and that |a1|=|a2| (1 = highest point in the path, 2 = lowest point)?

By the way, since this path is an arc, then ##|V| = ωL##
 
  • #4
babaliaris said:
PS: O MY GOD, I just figured something out... Well if you see the pendulum in a clock, you see it going up and down (oscillating) at the same speed! This means that when the pendulum reaches its maximum height it changes direction instantly! This is why the speed never changes but the direction of V changes!

This is the wrong assumption that I did and used V = 0 at the highest point which is not correct!
You've gone off the rails here. The speed is not constant and the speed is zero at the maximum height.

babaliaris said:
And the problem is, if V does not change in speed, how can it become zero? How can the pendulum oscillate without the speed ever becoming zero at the maximum angle?
The speed is zero but the acceleration is nonzero.

babaliaris said:
Using everything I learned from this post, I found that [...]
This is not the correct way to explain what you're doing. You need to provide the details as you go, just like textbook authors do when they solve worked examples.
 
  • Like
Likes Lnewqban
  • #5
My solution is in the pdf I attached. I don't know how to write everything in latex this is why I uploaded the pdf.
 
  • #6
It is not an SHM problem. Since the magnitude of the angle is initially unknown, you cannot assume the small angle approximation.
Just consider the two positions, maximum amplitude and zero amplitude.
In terms of ##\theta_{max}##, what is the linear acceleration at that angle (magnitude and direction)?
In terms of ##v_{max}##, what is the linear acceleration at zero angle (magnitude and direction)?
What equation relates ##\theta_{max}## and ##v_{max}##?
 
  • Like
Likes PeroK
  • #7
babaliaris said:
PS: O MY GOD, I just figured something out... Well if you see the pendulum in a clock, you see it going up and down (oscillating) at the same speed! This means that when the pendulum reaches its maximum height it changes direction instantly! This is why the speed never changes but the direction of V changes!
Is that what you see in the animation of post #21 of your previous thread?

https://www.physicsforums.com/threa...rce-how-to-calculate-the-full-vector.1045791/

I respectfully disagree with your teacher, evidently, you can describe it with math, but have not been able to see how it happens in real life.

:oops:
 
  • Like
Likes babaliaris
  • #8
haruspex said:
It is not an SHM problem. Since the magnitude of the angle is initially unknown, you cannot assume the small angle approximation.
Just consider the two positions, maximum amplitude and zero amplitude.
In terms of ##\theta_{max}##, what is the linear acceleration at that angle (magnitude and direction)?
In terms of ##v_{max}##, what is the linear acceleration at zero angle (magnitude and direction)?
What equation relates ##\theta_{max}## and ##v_{max}##?
Why are you ignoring the centripetal acceleration?
 
  • #9
  • Like
Likes babaliaris
  • #10
babaliaris said:
But now, how can I use the equation ##|a| = \sqrt{a_{T}^{2} + a_{N}^{2}} = \sqrt{g^{2}sin^{2}(θ(t)) + \frac{|V|^4}{L^{2}}}## and that |a1|=|a2| (1 = highest point in the path, 2 = lowest point)?

By the way, since this path is an arc, then ##|V| = ωL##
The fundamental observations you are missing are:

1) At the highest point, the bob is instantaneously at rest. The acceleration, therefore, is entirely tangential to the path. Otherwise, the bob would leave the circular path as it starts its downward swing. In other words, there is no centripetal acceleration at this point, owing to its zero velocity.

2) At the lowest point, the only forces (tension and gravity) are vertical, so there is no tangential acceleration. The acceleration at this point is entirely centripetal.
 
  • Like
Likes Lnewqban and babaliaris
  • #11
I thought of something else but I got a math error:

V at the lowest point:
##p1+k1 = p2 + k2 <=> mgL + 0 = 0 + \frac{1}{2}mV^2 <=> v = \sqrt{2gL}##

So at the highest point I now that: ##V = 0##, ##Θ(t) = Θ' = max{Θ(t)}##
and for the lowest point I know that: ##Θ(t) = 0## and ##v = \sqrt{2gL}##

and knowing from my prev calculations that ##|a| = \sqrt{g^2 sin^2 (θ(t)) + \frac{ (|V|^2)^2}{L^2}}##

so ##|a1| = \sqrt{ g^2 sin^2 (Θ') + 0 }##
##|a2| = \sqrt{ g^2 sin^2 (0) + \frac{ (\sqrt{2gL}^2)^2}{L^2} }##
##|a2| = \sqrt{ g^2 sin^2 (0) + \frac{ 4g^2L^2}{L^2} }##
##|a2| = \sqrt{ g^2 sin^2 (0) +4g^2 }##
##|a2| = \sqrt{ 4g^2 }##

so ##|a1|=|a2| <=> \sqrt{ g^2 sin^2 (Θ')} = \sqrt{ 4g^2 } <=> g^2 sin^2 (Θ') = 4g^2##
##<=>sin^2 (Θ') = 4 <=> sin(Θ') = 2 <=>Θ' = sin^{-1}(2) ##

but ##sin^{-1}(x)## is defined only for ##|x| < 1## i believe. The calculator gives an error for ##sin^{-1}(2)##
 
  • Sad
Likes PeroK
  • #12
PeroK said:
@babaliaris not the easiest pendulum problem!
Off Topic: Yes. By the way, this is first-year university physics. There were so many students that were cheating in the past, so the professor snapped and started creating some really difficult problems. This is actually one of the easiest problems he had in exams. I'm a fifth year and I know calc 1, 2, and complex analysis and still, I can't solve his problems. First-year students are probably depressed by now :p 90% of them fail with 2-3 out of 10 (marks).
 
  • #13
babaliaris said:
##p1+k1 = p2 + k2 <=> mgL + 0 = 0 + \frac{1}{2}mV^2 <=> v = \sqrt{2gL}##
How do you get mgL for the PE at the highest point it reaches, ##\theta_{max}##?
 
  • Like
Likes babaliaris
  • #14
I thought it didn't matter, so I assumed θ=90 where the m mass is indeed L meters from the ground.
So this is a mistake?

Yes at θmax I don't know the actual height so I can't find the potential energy there.

So in order for this to be right, I have to say:
##mgz = \frac{1}{2}mV^2## where z is the height when the pendulum is at ##Θ_{max}##?
 
  • #15
babaliaris said:
I thought of something else but I got a math error:

V at the lowest point:
##p1+k1 = p2 + k2 <=> mgL + 0 = 0 + \frac{1}{2}mV^2 <=> v = \sqrt{2gL}##

So at the highest point I now that: ##V = 0##, ##Θ(t) = Θ' = max{Θ(t)}##
and for the lowest point I know that: ##Θ(t) = 0## and ##v = \sqrt{2gL}##

and knowing from my prev calculations that ##|a| = \sqrt{g^2 sin^2 (θ(t)) + \frac{ (|V|^2)^2}{L^2}}##

so ##|a1| = \sqrt{ g^2 sin^2 (Θ') + 0 }##
##|a2| = \sqrt{ g^2 sin^2 (0) + \frac{ (\sqrt{2gL}^2)^2}{L^2} }##
##|a2| = \sqrt{ g^2 sin^2 (0) + \frac{ 4g^2L^2}{L^2} }##
##|a2| = \sqrt{ g^2 sin^2 (0) +4g^2 }##
##|a2| = \sqrt{ 4g^2 }##

so ##|a1|=|a2| <=> \sqrt{ g^2 sin^2 (Θ')} = \sqrt{ 4g^2 } <=> g^2 sin^2 (Θ') = 4g^2##
##<=>sin^2 (Θ') = 4 <=> sin(Θ') = 2 <=>Θ' = sin^{-1}(2) ##

but ##sin^{-1}(x)## is defined only for ##|x| < 1## i believe. The calculator gives an error for ##sin^{-1}(2)##
You are confusing this problem with the earlier one. In that problem the pendulum starts at the horizontal position and the initial potential energy is ##U=mgL.## Here, the pendulum starts at angle ##\theta_{\text{max}}.## What is the potential energy?

On edit: I see that @haruspex already asked the same question. Draw a triangle and find the height. It's the length ##L## minus the vertical length from the support point to the mass.
 
  • Like
Likes babaliaris
  • #16
kuruman said:
Here, the pendulum starts at angle ##\theta_{\text{max}}.## What is the potential energy?
##P_{@Θ_{max}} = mgz## where z = the distance from the ground up to the mass m, and z is uknown...
 
  • #17
babaliaris said:
##P_{@Θ_{max}} = mgz## where z = the distance from the ground up to the mass m, and z is uknown...
See my edited post about finding the height.
 
  • Like
Likes babaliaris
  • #18
Found it ##z = L[1-cosθ']## where ##θ' = θ_{max}##

1664211385038.jpg

1664211385032.jpg


I ended up to ##sinθ' + 2cos(θ') = 2##. Unfortunately, I don't remember how to solve that. I always forget trig formulas...

Edited:
Capture.PNG
 
Last edited:
  • #19
Use the trig identities of the half angle in the second equation from the bottom
##\sin\theta=2\sin(\frac{\theta}{2})\cos(\frac{\theta}{2})##
##1-\cos\theta=2\sin^2(\frac{\theta}{2})##
 
  • Like
Likes babaliaris
  • #20
Or, just square the equation$$\sin \theta = 2(1-\cos \theta)$$ and use ##\sin^2 \theta + \cos^2 \theta = 1##. That's one formula you should never forget!
 
  • Like
Likes Lnewqban
  • #21
So in conclusion, Is this the correct solution? Did I make it!? Thank you for your time and guidance!
 
  • #22
babaliaris said:
So in conclusion, Is this the correct solution? Did I make it!? Thank you for your time and guidance!
What solution? Note that ##\theta = 0## is a degenerate solution.
 
  • Like
Likes babaliaris
  • #23
All the math and physics that I did to reach in the equation ##sinθ' + 2cos(θ') = 2##
 
  • #24
babaliaris said:
All the math and physics that I did to reach in the equation ##sinθ' + 2cos(θ') = 2##
That's correct, but it's not what I'd call a solution.
 
  • Like
Likes babaliaris
  • #25
babaliaris said:
All the math and physics that I did to reach in the equation ##sinθ' + 2cos(θ') = 2##
You still have to solve it and find a numerical value for ##\theta_{\text{max}}.##

Actually, this is a nice problem. It's the first time I see it.
 
  • Like
Likes PeroK and babaliaris
  • #26
I will do it tomorrow since I'm exhausted. I'm struggling with that problem for 3 days now. Never in my life, I had to deal with a problem for 3 days continuously. I'm glad that I finally figured it out!

How did great scientists try to solve a problem for years and not get crazy?
 
  • #27
With the benefit of hindsight there is an easier way to solve this.
Just recognize it's a 3-4-5 triangle with ##\cos\theta=\dfrac{3}{5}~;~\sin\theta=\dfrac{4}{5}.##
 
  • Like
Likes Lnewqban

What is a pendulum?

A pendulum is a weight suspended from a pivot point that can swing back and forth due to the force of gravity.

How do you calculate the acceleration of a pendulum?

The acceleration of a pendulum can be calculated using the formula a = g * sin(theta), where g is the acceleration due to gravity (9.8 m/s^2) and theta is the angle of the pendulum.

What is the maximum angle of a pendulum?

The maximum angle of a pendulum is when it swings to its highest point, also known as the amplitude. This angle is usually less than 90 degrees, depending on the length of the pendulum.

How does the acceleration affect the maximum angle of a pendulum?

The acceleration of a pendulum is directly proportional to the maximum angle. This means that as the acceleration increases, so does the maximum angle of the pendulum.

Why is it important to know the maximum angle of a pendulum?

Knowing the maximum angle of a pendulum is important because it can help predict the motion and behavior of the pendulum. It also allows for the calculation of other important factors, such as the period and frequency of the pendulum's swing.

Similar threads

  • Introductory Physics Homework Help
2
Replies
38
Views
1K
  • Introductory Physics Homework Help
Replies
29
Views
984
  • Introductory Physics Homework Help
Replies
2
Views
294
  • Introductory Physics Homework Help
Replies
15
Views
983
  • Introductory Physics Homework Help
Replies
20
Views
976
  • Introductory Physics Homework Help
Replies
2
Views
844
  • Introductory Physics Homework Help
Replies
9
Views
636
  • Introductory Physics Homework Help
Replies
5
Views
2K
  • Introductory Physics Homework Help
2
Replies
35
Views
2K
  • Introductory Physics Homework Help
Replies
4
Views
166
Back
Top